Economy Prelims Pyqs Since Mid-1990s 3 of 5 Meritnest

Download as pdf or txt
Download as pdf or txt
You are on page 1of 38

Economy PYQs 3 of 5 External Sector, International Institutions

Note: (d) World Bank

#1: This PDF is intended solely for personal use. We 3. Consider the following: [2022]
kindly ask you to avoid unauthorised sharing and
support our affordable initiatives. If you are aware (1) Asian Infrastructure Investment Bank
of anyone who might benefit from these efforts,
please explore our Refer and Earn programme. For (2) Missile Technology Control Regime
those who know at least 25 individuals who could
benefit from our initiatives, we encourage you to (3) Shanghai Cooperation Organisation
consider our partner programme. Our referral and
partner benefits are among the best in the industry. India is a member of which of the above?
oslo
#2: We have deliberately not included questions (a) 1 and 2 only
from the 2023 Prelims paper in this question bank. (b) 3 only
Once you've covered all the questions (Level-1, (c) 2 and 3 only
Level-2, and PYQs) and handouts for all subjects,
(d) 1, 2 and 3
please consider attempting the 2023 question
paper.
4. Consider the following statements. [2021]
#3: To report inaccuracies, if any, please email us at
[email protected]. (1) Foreign currency convertible bonds

#4: You can check corrections to this PDF from the (2) Foreign institutional investment with certain
page you downloaded it from. conditions

#5: We will also communicate all corrections via (3) Global depository receipts
email ~1 month prior to the upcoming Prelims to
everyone who downloaded this PDF from (4) Non-resident external deposits
https://www.meritnest.com
Which of the above can be included in Foreign
Direct Investments?
1. Which one of the following situations best (a) 1, 2 and 3
reflects “Indirect Transfers” often talked about in [email protected]
(b) 3 only
media recently with reference to India? [2022]
(c) 2 and 4 only
(a) An Indian company investing in a foreign
(d) 1 and 4 only
enterprise and paying taxes to the foreign country
on the profits arising out of its investment
5. Consider the following statements. [2021]
(b) A foreign company investing in India and paying
taxes to the country of its base on the profits arising
The effect of devaluation of a currency is that it
out of its investment
necessarily
(c) An Indian company purchases tangible assets in
9654469135
a foreign country and sells such assets after their (1) Improves the competitiveness of the domestic
value increases and transfers the proceeds to India exports in the foreign markets.
(d) A foreign company transfers shares and such
shares derive their substantial value from assets (2) Increases the foreign value of domestic
located in India currency.

2. Rapid Financing Instrument and Rapid Credit (3) Improves the trade balance.
Facility are related to the provisions of lending by
which one of the following? [2022] Which of the above statements is/are correct?

(a) Asian Development Bank (a) 1 only


(b) International Monetary Fund (b) 1 and 2 only
(c) United Nations Environment Programme (c) 3 only
Finance Initiative (d) 2 and 3 only

https://www.meritnest.com Page 1
Economy PYQs 3 of 5 External Sector, International Institutions

(4) India suffers from an overall trade/current


6. If another global financial crisis happens in the account deficit.
near future, which of the following actions/policies
are most likely to give some immunity to India? Select the correct answer using the code given
oslo
[2020] 9654469135 below:
(a) 1 and 2 only
(1) Not depending on short-term foreign
borrowings (b) 2 and 4 only
(c) 3 only
(2) Opening up to more foreign banks (d) 1, 3 and 4 only

(3) Maintaining full capital account convertibility


10. With reference to Trade-Related Investment
[email protected]
Measures (TRIMS), which of the following
Select the correct answer using the code given
statements is/are correct? [2020]
below:
(a) 1 only (1) Quantitative restrictions on imports by foreign
(b) 1 and 2 only investors are prohibited.
(c) 3 only
(2) They apply to investment measures related to
(d) 1, 2 and 3 trade in both goods and services.

7. “Gold Tranche” (Reserve Tranche) refers to (3) They are not concerned with the regulation of
[2020] foreign investment.
(a) a loan system of the World Bank
Select the correct answer using the code given
(b) one of the operations of a Central Bank below:
(c) a credit system granted by WTO to its members
(a) 1 and 2 only
(d) a credit system granted by IMF to its members
(b) 2 only
(c) 1 and 3 only
8. With reference to Foreign Direct Investment in
India, which one of the following is considered its (d) 1, 2 and 3
major characteristic? [2020]
11. Which one of the following is not the most
(a) It is the investment through capital instruments
likely measure the Government/RBI takes to stop
essentially in a listed company.
the slide of Indian rupee? [2019]
(b) It is a largely non-debt creating capital flow.
(a) Curbing imports of non-essential goods and
(c) It is the investment which involves
promoting exports.
debt-servicing.
(b) Encouraging Indian borrowers to issue rupee
(d) It is the investment made by foreign institutional
denominated Masala Bonds.
investors in the Government securities.
(c) Easing conditions relating to external
commercial borrowing.
9. With reference to the international trade of
India at present, which of the following statements (d) Following an expansionary monetary policy.
is/are correct? [2020]
12. Which one of the following is not a sub-index
(1) India’s merchandise exports are less than its of the World Bank’s ‘Ease of Doing Business
merchandise imports. Index’? [2019]
(a) Maintenance of law and order
(2) India’s imports of iron and steel, chemicals,
fertilisers and machinery have decreased in recent (b) Paying taxes
years. (c) Registering property
(d) Dealing with construction permits
(3) India’s exports of services are more than its
imports of services.
13. The Global Competitiveness Report is
published by the [2019]

https://www.meritnest.com Page 2
Economy PYQs 3 of 5 External Sector, International Institutions

(a) International Monetary Fund (a) World Economic Forum


(b) United Nations Conference on Trade and (b) UN Human Rights Council
Development (c) UN Women
(c) World Economic Forum (d) World Health Organisation
(d) World Bank
18. Recently, which one of the following
[email protected]
14. India enacted the Geographical Indications of currencies has been proposed to be added to the
Goods (Registration and Protection) Act, 1999 in basket of IMF’s SDR? [2016]
order to comply with the obligations to [2018]
(a) Rouble
(a) ILO (b) Rand
(b) IMF (c) Indian Rupee
(c) UNCTAD (d) Renminbi
(d) WTO
19. Which of the following best describes the term
15. Consider the following statements. [2017] ‘import cover’, sometimes seen in the news? [2016]
(a) It is the ratio of value of imports to the Gross
(1) India has ratified the Trade Facilitation
Domestic Product of a country.
Agreement (TFA) of WTO.
(b) It is the total value of imports of a country in a
(2) TFA is a part of WTO’s Bali Ministerial Package year.
of 2013. (c) It is the ratio between the value of exports and
that of imports between two countries.
(3) TFA came into force in January 2016. (d) It is the number of months of imports that could
be paid for by a country’s international reserves.
Which of the statements given above is/are
correct?
20. ‘Global Financial Stability Report’ is prepared
(a) 1 and 2 only by the [2016]
(b) 1 and 3 only (a) European Central Bank
(c) 2 and 3 only (b) International Monetary Fund
(d) 1, 2 and 3 (c) International Bank for Reconstruction and
Development
16. With reference to the 'National Intellectual (d) Organisation for Economic Cooperation and
Property Rights Policy', consider the following Development
statements: [2017]
oslo
21. In the context of which of the following do you
(1) It reiterates India’s commitment to the Doha
sometimes find the terms ‘amber box, blue box
Development Agenda and the TRIPS Agreement.
and green box’ in the news? [2016]
(2) Department of Industrial Policy and Promotion (a) WTO affairs
is the nodal agency for regulating intellectual (b) SAARC affairs
property rights in India.
(c) UNFCCC affairs
Which of the above statements is/are correct? (d) India-EU negotiations on FTA

(a) 1 only
22. With reference to the International Monetary
(b) 2 only and Financial Committee (IMFC), consider the
(c) Both 1 and 2 9654469135 following statements: [2016]
(d) Neither 1 nor 2
(1) IMFC discusses matters of concern affecting the
global economy, and advises the International
17. Which of the following gives 'Global Gender
Monetary Fund (IMF) on the direction of its work.
Gap Index' ranking to the countries of the world?
[2017]

https://www.meritnest.com Page 3
Economy PYQs 3 of 5 External Sector, International Institutions

(2) The World Bank participates as observer in (d) United Nations Environment Programme
IMFC’s meetings.
27. Which of the following organizations brings
Which of the statements given above is/are
out the publication known as 'World Economic
correct?
Outlook'? [2014]
(a) 1 only
(a) The International Monetary Fund
(b) 2 only
(b) The United Nations Development Programme
(c) Both 1 and 2
(c) The World Economic Forum
(d) Neither 1 nor 2
(d) The World Bank
oslo
23. Convertibility of rupee implies: [2015]
28. The balance of payments of a country is a
(a) Being able to convert rupee notes into gold. systematic record of [2013]
(b) Allowing the value of rupee to be fixed by (a) All import and export transactions of a country
market forces. during a given period of time, normally a year.
(c) Freely permitting the conversion of rupee to (b) Goods exported from a country during a year.
other currencies and vice versa.
(c) Economic transaction between the governments
(d) Developing an international market for of one country to another.
currencies in India.
(d) Capital movements from one country to
another.
24. ‘Basel III Accord’ or simply ‘Basel III’, often
seen in the news, seeks to [2015]
29. Which one of the following groups of items is
(a) develop national strategies for the conservation included in India’s Foreign Exchange Reserves?
and sustainable use of biological diversity. [2013]
(b) improve banking sector’s ability to deal with (a) Foreign-currency assets, Special Drawing Rights
financial and economic stress and improve risk (SDRs) and loans from foreign countries.
management.
(b) Foreign-currency assets, gold holdings of the
(c) reduce the greenhouse gas emissions but places RBI and SDRs.
a heavier burden on developed countries.
(c) Foreign currency assets, loans from the World
(d) transfer technology from developed countries to Bank and SDRs. [email protected]
poor countries to enable them to replace the use of
(d) Foreign currency assets, gold holdings of the RBI
chlorofluorocarbons in refrigeration with harmless
and loans from the World Bank.
chemicals.

30. Consider the following statements: [2012]


25. Which one of the following issues the “Global
Economic Prospects” report periodically? [2015]
The price of any currency in international market is
(a) The Asian Development Bank decided by the:
(b) The European Bank for Reconstruction and
9654469135
Development (1) World Bank
(c) The US Federal Reserve Bank
(2) demand for goods/services provided by the
(d) The World Bank country concerned

26. The terms ‘Agreement on Agriculture’, (3) stability of the government of the concerned
‘Agreement on the Application of Sanitary and country
Phytosanitary Measures’ and 'Peace Clause'
appear in the news frequently in the context of the (4) economic potential of the country in question
affairs of the [2015]
Which of the statements given above are correct?
(a) Food and Agriculture Organization
(b) United Nations Framework Conference on (a) 1, 2, 3 and 4
Climate Change (b) 2 and 3 only
(c) World Trade Organization (c) 3 and 4 only

https://www.meritnest.com Page 4
Economy PYQs 3 of 5 External Sector, International Institutions

(d) 1 and 4 only (d) Neither 1 nor 2

31. Consider the following actions which the 34. The SEZ Act, 2005 which came into effect in
Government can take: [2011] February
oslo
2006 has certain objectives. In this
9654469135 context, consider the following: [2010]
(1) Devaluing the domestic currency.
(1) Development of infrastructure facilities.
(2) Reduction in the export subsidy.
(2) Promotion of investment from foreign sources.
(3) Adopting suitable policies which attract greater
FDI and more funds from FIIs. (3) Promotion of exports of services only.

[email protected]
Which of the above action/actions can help in Which of the above are the objectives of this Act?
reducing the current account deficit? (a) 1 and 2 only
(a) 1 and 2 only (b) 3 only
(b) 2 and 3 only (c) 2 and 3 only
(c) 3 only (d) 1, 2 and 3
(d) 1 and 3 only
35. Which of the following is/are treated as
32. In the context of Indian economy, consider the artificial currency? [2010]
following statements: [2011] (a) ADR
(b) GDR
(1) The growth rate of GDP has steadily increased in
the last five years. (c) SDR
(d) Both ADR and SDR
(2) The growth rate in per capita income has
steadily increased in the last five years. 36. In order to comply with TRIPS Agreement,
India enacted the Geographical Indications of
Which of the statements given above is/are Goods (Registration & Protection) Act, 1999. The
correct? difference/differences between a "Trade Mark"
(a) 1 only and a Geographical Indication is/are: [2010]
(b) 2 only
(1) A Trade Mark is an individual or a company’s
(c) Both 1 and 2 right whereas a Geographical Indication is a
(d) Neither 1 nor 2 community’s right.

33. Why is the offering of "teaser loans" by (2) A Trade Mark can be licensed whereas a
commercial banks a cause of economic concern? Geographical Indication cannot be licensed.
[2011]
(3) A Trade Mark is assigned to the manufactured
(1) The teaser loans are considered to be an aspect goods whereas the Geographical Indication is
of sub-prime lending and banks may be exposed to assigned to the agricultural goods/products and
the risk of defaulters in future. handicrafts only.

(2) In India, the teaser loans are mostly given to Which of the statements given above is/ are
inexperienced entrepreneurs to set up correct?
manufacturing or export units. (a) 1 only
(b) 1 and 2 only
Which of the statements given above is/are
correct? (c) 2 and 3 only
(d) 1, 2 and 3
(a) 1 only
(b) 2 only
37. Consider the following countries: [2010]
(c) Both 1 and 2

https://www.meritnest.com Page 5
Economy PYQs 3 of 5 External Sector, International Institutions

(1) Brazil 42. NAMA-11 group of countries frequently


appears in the news in the context of the affairs of
9654469135
(2) Mexico which one of the following? [2009]
(a) Nuclear Suppliers Group
(3) South Africa
(b) World Bank
According to UNCTAD, which of the above is/are (c) World Economic Forum
categorized as "Emerging Economies"? (d) W.T.O.
[email protected]
(a) 1 only
(b) 1 and 3 only 43. Who among the following is the founder of
World Economic Forum? [2009]
(c) 2 and 3 only
(d) 1, 2 and 3 (a) Klaus Schwab
(b) John Kenneth Galbraith
38. The International Development Association, a (c) Robert Zoellick
lending agency, is administered by the [2010] (d) Paul Krugman
(a) International Bank for Reconstruction and
Development 44. With reference to the United Nations, consider
(b) International Fund for Agricultural the following statements: [2009]
Development
(1) The Economic and Social Council (ECOSOC) of
(c) United Nations Development Programme
UN consists of 24 member States.
(d) United Nations Industrial Development
Organization (2) Member states are elected by a 2/3rd majority
of the General Assembly for a 3-year term.
39. As regards the use of international food safety
standards as reference point for the dispute Which of the above statements is/are correct?
settlements, which one of the following does WTO
(a) 1 only
collaborate with? [2010]
(b) 2 only
(a) Codex Alimentarius Commission
(c) Both 1 and 2
(b) International Federation of Standards Users
(d) Neither 1 nor 2
(c) International Organization for Standardization
(d) World Standards Cooperation 45. Consider the following pairs:

40. In the context of the affairs of which of the Large Bank -oslo
Country of origin [2009]
following is the phrase "Special Safeguard
Mechanisms" mentioned in the news frequently? (1) ABN Amro Bank - USA
[2010]
(2) Barclays Bank - UK
(a) United Nations Environment Programme
(3) Kookmin Bank - Japan
(b) World Trade Organization
(c) ASEAN - India Free Trade Agreement Which of the above pairs is/are correctly matched?
(d) G-20 Summits (a) 1 only
(b) 2 only
41. Other than Venezuela, which one among the
(c) 1 and 2
following from South America is a member of
OPEC? [2009] (d) 2 and 3

(a) Argentina
46. Consider the following pairs:
(b) Brazil
(c) Ecuador Organisation - Location of Headquarters [2009]
(d) Bolivia
(1) Asian Development Bank - Tokyo

https://www.meritnest.com Page 6
Economy PYQs 3 of 5 External Sector, International Institutions

(2) Asia-Pacific Economic Cooperation - Singapore (c) Direct tax levied by the State Government
(3) Association of South East Asian Nations - (d) Indirect tax levied by the State Government
Bangkok
51. Consider the following statements: [2006]
Which of the above pairs is/are correctly matched?
(1) In India, during the financial year 2004 – 2005,
(a) 1 only
an increase of below 10% over the value of exports
(b) 2 only (in rupee terms) in the financial year 2003 – 2004
(c) Both 1 and 2 was reported.
(d) Neither 1 nor 2
(2) According to the WTO, India’s share in the world
oslo
merchandise exports crossed 2% in the year 2005.
47. India is a member of which of the following?
[2008]
Which of the statement(s) given above is/are
correct?
(1) Asian Development Bank
(a) 1 only
(2) Asia-Pacific Economic Cooperation (b) 2 only
(c) Both 1 and 2
(3) Colombo Plan
(d) Neither 1 nor 2
(4) Organization for Economic Cooperation and
Development (OECD) 52. Which one of the following companies has
started a rural marketing network called
Select the correct answer using the codes given ‘e-chaupals’? [2005]
below:
(a) ITC
(a) 1 and 3 only (b) Dabur
(b) 2 and 4 only (c) Proctor & Gamble
(c) 1, 2 and 3 only (d) Hindustan Lever
(d) 1, 2, 3 and 4
53. Consider the following statements: [2005]
48. Tarapore Committee was associated with [email protected]
which one of the following? [2007] (1) The number of post offices in India is in excess of
1.5 lakhs.
(a) Special Economic Zones
(b) Fuller capital account convertibility (2) Bharat Sanchar Nigam Limited (BSNL) was
(c) Foreign exchange reserves formed in the year 1997.
(d) Effect of oil-prices on the Indian economy
(3) Telecom Regulatory Authority of India (TRAI)
was established in the year 2000.
49. Who among the following served as the Chief
9654469135
Economist of the International Monetary Fund?
Which of the statements given above is/are
[2007]
correct?
(a) Ashok Lahiri
(a) 1, 2 and 3
(b) Sumantra Ghoshal
(b) 1 and 2
(c) Saumitra Chaudhuri
(c) 1 only
(d) Raghuram Rajan
(d) 3 only

50. Which one of the following is the correct


54. Assertion (A): For the first time, India had no
statement? [2006]
trade deficit in the year 2002- 2003.
Service tax is a/an:
Reason (R): For the first time, India’s exports
(a) Direct tax levied by the Central Government
crossed worth $50 billion in the year 2002-03.
(b) Indirect tax levied by the Central Government [2004]

https://www.meritnest.com Page 7
Economy PYQs 3 of 5 External Sector, International Institutions

Codes:
(a) Both A and R are individually true and R is the
correct explanation of A ABCD
(b) Both A and R are individually true but R is not (a) 2 3 4 1
the correct explanation of A
(b) 4 1 2 3
(c) A is true but R is false
(c) 2 1 4 3
(d) A is false but R is true
(d) 4 3 2 1

55. Consider the following statements: India


57. Consider the following given statements:
continues to be dependent on imports to meet the
[2003]
requirement of oilseeds in the country because:
oslo
[2004]
(1) The maximum limit of shareholding of Indian
promoters in private sector banks in India is 49
(1) Farmers prefer to grow food grains with highly
percent of the paid-up capital.
remunerative support prices.
(2) Foreign Direct Investment up-to 49 percent
(2) Most of the cultivation of oilseed crops
from all sources is permitted in private sector banks
continues to be dependent on rainfall.
in India under the automatic route.
(3) Oils from the seeds of tree origin and rice bran
Which of these statements is/are correct?
have remained unexploited.
(a) 1 only
(4) It is far cheaper to import oilseeds than to (b) 2 only
cultivate the oilseed crops.
(c) Both 1 and 2
Which of the statements given above is/are (d) Neither 1 nor 2
correct?
58. Assertion (A): The new EXIM policy is liberal,
(a) 1 and 2
market-oriented and favours global trade.
(b) 1, 2 and 3
(c) 3 and 4 Reason (R): GATT has played a significant role in the
(d) 1, 2, 3 and 4 liberalisation of the economy. [2003]
[email protected]
(a) Both A and R are individually true and R is the
56. Match List-I (Agency) with List-II correct explanation of A
(Headquarters) and select the correct answer (b) Both A and R are individually true but R is not
using the codes given below the Lists: [2004] the correct explanation of A
(c) A is true but R is false
List-I (Agency)
(d) A is false but R is true
(A) United Nations Development Programme
(UNDP)
59. With reference to India, which one of the
9654469135
(B) United Nations Environment Programme following statements is NOT correct? [2003]
(UNEP)
(a) IPCL is India’s largest petrochemical company
(C) United Nations Industrial Development (b) RIL is the largest private sector company in India
Organization (UNIDO)
(c) MTNL is listed on NYSE
(D) Universal Postal Union (UPU) (d) BSNL is the first telecom service organization in
India to launch a nationwide cellular service at one
List-II (Headquarters) time
1. Nairobi
60. The government holding in BPCL is: [2003]
2. Vienna
(a) more than 70%
3. Berne
(b) between 60% and 70%
4. New York
(c) between 50% and 60%

https://www.meritnest.com Page 8
Economy PYQs 3 of 5 External Sector, International Institutions

(d) less than 50% (4) Least developed country members of WTO are
not required to apply for the provisions of TRIPS
9654469135
61. India’s external debt increased from US Agreements for a period of 20 years from the
$98,158 million as at the end of March 2000 to US general date of application of the Agreement
$100,225 million as at the end of March 2001 due
to increase in: [2002] Which of these statements are correct?

(a) multilateral and bilateral debt (a) 1, 2, 3 and 4


(b) rupee debt (b) 2, 3 and 4
[email protected]

(c) commercial borrowings and NRI deposits (c) 1, 2 and 4


(d) borrowing from International Monetary Fund (d) 1 and 3

62. Global capital-flows to developing countries 65. Match List-I (Organisation) with List-II
increased significantly during the nineties. In view (Headquarters) and select the correct answer
of the East Asian financial crisis and Latin using the codes given below the Lists: [2002]
American experience, which type of inflow is good
for the host country? [2002] List-I (Organisation)

(a) Commercial loans (A) International Atomic Energy Agency


(b) Foreign Direct Investment (B) International Telecommunication Union
(c) Foreign Portfolio Investment (C) Council of the European Union
(d) External Commercial Borrowings
(D) Organisation for Economic Cooperation and
Development
63. Consider the following statements: [2002]
List-II (Headquarters)
Full convertibility of the rupee may mean
1. Brussels
(1) its free float with other international currencies. 2. Geneva

(2) its direct exchange with any other international 3. Paris


currency at any prescribed place inside and outside 4. Vienna
the country.
Codes:
(3) it acts just like any other international currency.
ABCD
Which of these statements are correct? (a) 1 2 4 3 oslo
(a) 1 and 2 (b) 4 2 1 3
(b) 1 and 3 (c) 4 2 1 3
(c) 2 and 3 (d) 4 2 1 3
(d) 1, 2 and 3
66. The Asia Pacific Economic Cooperation Forum
64. Consider the following statements: [2002] was held in the year 2001 in: [2002]
(a) Bangkok
(1) The World Intellectual Property Organisation
(b) Hongkong
(WIPO) is a specialised agency of United Nations
System of Organisations (c) Jakarta
(d) Shanghai
(2) WIPO has its headquarters at Rome
67. Consider the following financial institution of
(3) The Trade Related Aspects of Intellectual India: [2002]
Property Rights (TRIPS) Agreement is binding on all
WTO members (1) Industrial Finance Corporation of India (IFCI)

https://www.meritnest.com Page 9
Economy PYQs 3 of 5 External Sector, International Institutions

(2) Industrial Credit and Investment Corporation of


I. labour-related issues.
India (ICICI)
II. environment related issues.
(3) Industrial Development Bank of India (IDBI)
III. terrorism related issues.
(4) National Bank for Agriculture and Rural
Development (NABARD)
IV. debt-related issues.
The correct chronological sequence of the
Which of these statements are correct?
establishment of these institutions is:
(a) I, III and IV
(a) 1, 2, 3, 4
(b) I and II
(b) 2, 3, 4, 1
(c) II and III
(c) 3, 4, 1, 2
(d) II and IV
(d) 4, 1, 2, 3

71. Match List I with List II and select the correct


68. Assertion (A): Ceiling on foreign exchange for a
answer using the codes given below the Lists:
host of current account transaction heads was
[email protected] [1999]
lowered in the year 2000.
List-I
Reason (R): There was a fall in foreign currency
assets also. [2001] I. WTO
(a) Both A and R are individually true, and R is the II. IDA
correct explanation of A
III. IMF
(b) Both A and R are individually true, but R is NOT a
correct explanation of A IV. IBRD
(c) A is true, but R is false
List-II
(d) A is false, but R is true
A) Provides loans to address short-term balance of
payment problems
69. Consider the following organisations: [2001]
B) Multi-lateral trade negotiation body
I. International Bank for Reconstruction and
C) Sanction of soft loans
Development
D) Facilitating lending and borrowing for
II. International Finance Corporation reconstruction and development

III. International Fund for Agricultural Codes:


Development
(a) I-B, II-C, III-D, IV-A
IV. International Monetary Fund (b) I-B, II-C, III-A, IV-D
9654469135
(c) I-C, II-B, III-D, IV-A
Which of these are the agencies of the United (d) I-C, II-B, III-A, IV-D
Nations?
(a) I and II 72. Consider the following statements: [1998]
(b) II and III
The price of any currency in international market is
(c) III and IV
decided by the
(d) I, II, III and IV
(I) World Bank.
70. Consider the following statement: [2000]
(II) demand for goods/services provided by the
The Ministerial Meeting of the WTO held in country concerned.
December 1999 was unsuccessful oslobecause it
attempted to link trade with:

https://www.meritnest.com Page 10
Economy PYQs 3 of 5 External Sector, International Institutions

(III) stability of the government of the concerned


country. 76. Consider the following items imported by
India: [1996]
(IV) economic potential of the country.
(I) Capital goods
Of these statements:
(II) Petroleum
(a) I, II, III and IV are correct
[email protected]
(b) II and III are correct (III) Pearls and precious stones
(c) III and IV are correct
(d) I and IV are correct (IV) Chemicals

(V) Iron and Steel


73. Capital Account Convertibility of the Indian
Rupee implies [1998]
The correct sequence of the decreasing order of
(a) that the Indian Rupee can be exchanged by the these items, in terms of value, is
authorised dealers for travel
(a) I, II, III, IV, V
(b) that the Indian Rupee can be exchanged for any
(b) I, II, IV, III, V
major currency for the purpose of trade in goods
and services (c) II, I, III, IV, V
(c) that the Indian Rupee can be exchanged for any (d) II, I, IV, V, III
major currency for the purpose of trading financial
assets 77. Which of the following were the aims behind
(d) None of the above the setting up of the World Trade Organization?
[1996]
74. Assertion (A): The United States of America
I. Promotion of free trade and resource flows across
has threatened to ask the World Trade
countries.
Organisation (WTO) to apply sanctions against the
developing countries for the non-observance of
II. Protection of intellectual property rights.
ILO conventions.
III. Managing balanced trade between different
Reason (R): The United States of America itself has
countries.
adopted and implemented those ILO conventions.
[1998]
IV. Promotion of trade between the former East
(a) Both A and R are true, and R is the correct block countries and the western world.
explanation of A
(b) Both A and R are true, but R is NOToslo
a correct Select the correct answer by using the codes given
explanation of A below:
(c) A is true, but R is false
Code:
(d) A is false, but R is true
(a) I, II, III and IV
75. Hawala transactions relate to payments [1996] (b) I and II
(c) II and III
(a) received in rupees against overseas currencies
and vice-versa without going through the official (d) I and IV
channels
(b) received for sale/transfer of shares without 78. Consider the following statements: [1996]
going through the established stock exchanges
Most international agencies which fund
(c) received as commission for services rendered to
9654469135
development programmes in India on
overseas investors/buyers/sellers in assisting them
intergovernmental bilateral agreements, mainly
to get over the red tape and/or in getting
provide:
preferential treatment
(d) made to political parties or to individuals for I. Technical assistance.
meeting election expenses

https://www.meritnest.com Page 11
Economy PYQs 3 of 5 External Sector, International Institutions

II. Soft loans which are required to be paid back II. It will help boost exports of agricultural
with interest. commodities from India.
9654469135

III. Grants, not required to be paid back. III. India’s share in the world trade is likely to triple
by the year 2000 A.D.
IV. Food assistance to alleviate poverty.
Select the correct answer using the codes given
Choose the correct answer from the codes given below:
[email protected]
below:
Codes:
Codes:
(a) I, II and III
(a) II and IV are correct (b) I and II
(b) I, II and III are correct (c) I and III
(c) I, II and IV are correct (d) II and III
(d) III and IV are correct

79. The new Exim Policy announced in 1992, is for


a period of [1995]
(a) 3 years
(b) 4 years
(c) 7 years
(d) 5 years

80. Which of the following constitute the World


Bank? [1995]

I. International Bank for Reconstruction and


Development

II. International Finance Corporation

III. International Development Association

IV. International Monetary Fund


oslo

Choose the correct answer from the codes given


below:

Codes:
(a) I, II and III
(b) I and II
(c) III and IV
(d) I, II, III and IV

81. Which of the following benefits are likely to


accrue to India from the World Trade
Organisation? [1995]

I. India’s share in the world trade is to go up from


the present 600 million US dollars to 5 billion US
dollars by 2000 A.D.

https://www.meritnest.com Page 12
Economy PYQs 3 of 5 External Sector, International Institutions

(c) United Nations Environment Programme


Finance Initiative
9654469135
(d) World Bank

Solution 2. Correct Option: (b)


Option (b) is correct.

[email protected]
• The Rapid Financing Instrument (RFI) provides
rapid financial assistance to all member countries
facing a balance of payments crisis.

• It was part of a broader reform to make the


International Monetary Fund’s (IMF) financial
support more flexible and to address the diverse
needs of member countries, replacing the IMF’s
previous emergency assistance policy.

• Rapid Credit Facility (RCF) offers rapid


concessional financial assistance, generally to
low-income countries (LICs) facing a balance of
payments crisis without ex-post conditionality and
where a full-fledged economic program is neither
necessary nor feasible.

1. Which one of the following situations best • The RCF was created under the Poverty
reflects “Indirect Transfers” often talked about in Reduction and Growth Trust (PRGT) as part of a
media recently with reference to India? [2022] reform to make the Fund’s financial support more
flexible and better tailored to the diverse needs of
(a) An Indian company investing in a foreign
LICs, including in times of crisis.
enterprise and paying taxes to the foreign country
on the profits arising out of its investment
(b) A foreign company investing in India and paying 3. Consider the following: [2022]
taxes to the country of its base on the profits arising
out of its investment (1) Asian Infrastructure Investment Bank
(c) An Indian company purchases tangible assets in
(2) Missile Technology Control Regime
a foreign country and sells such assets after their
oslo
value increases and transfers the proceeds to India
(3) Shanghai Cooperation Organisation
(d) A foreign company transfers shares and such
shares derive their substantial value from assets India is a member of which of the above?
located in India
(a) 1 and 2 only
Solution 1. Correct Option: (d) (b) 3 only
(c) 2 and 3 only
Option (d) is correct
(d) 1, 2 and 3
• Indirect transfers involve the scenario where a
foreign company sells shares that derive significant Solution 3. Correct Option: (c)
value from assets located within India.
Option (c) is correct.

2. Rapid Financing Instrument and Rapid Credit • The Shanghai Cooperation Organisation (SCO)
Facility are related to the provisions of lending by currently comprises eight Member States: China,
which one of the following? [2022] India, Kazakhstan, Kyrgyzstan, Russia, Pakistan,
Tajikistan, and Uzbekistan.
(a) Asian Development Bank
(b) International Monetary Fund

https://www.meritnest.com Page 13
Economy PYQs 3 of 5 External Sector, International Institutions

• Missile Technology Control Regime (MTCR): On • Foreign Institutional Investors (FIIs) can invest up
27 June, India became a full member of the MTCR to 10% of the total paid-up capital of an Indian
after9654469135
a deal with Italy, marking India's first entry company.
into any multilateral export control regime.
• Indian companies can raise funds overseas
• Asian Infrastructure Investment Bank (AIIB): On through American Depository Receipt
24 October 2014, twenty-one countries, including (ADR)/Global Depository Receipts (GDRs) under
India, signed a Memorandum of Understanding specific government schemes.
[email protected]
(MOU) regarding the AIIB in Beijing, China, with
other member countries being Bangladesh, Brunei, • Non-resident external (NRE) deposits are not
Cambodia, Kazakhstan, Kuwait, Laos, Malaysia, classified as FDI.
Myanmar, Mongolia, Nepal, Oman, Pakistan,
Philippines, Qatar, Singapore, Sri Lanka, Thailand,
5. Consider the following statements. [2021]
Uzbekistan, and Vietnam.
The effect of devaluation of a currency is that it
4. Consider the following statements. [2021] necessarily

(1) Foreign currency convertible bonds (1) Improves the competitiveness of the domestic
exports in the foreign markets.
(2) Foreign institutional investment with certain
conditions (2) Increases the foreign value of domestic
currency.
(3) Global depository receipts
(3) Improves the trade balance.
(4) Non-resident external deposits
Which of the above statements is/are correct?
Which of the above can be included in Foreign
(a) 1 only
Direct Investments?
(b) 1 and 2 only
(a) 1, 2 and 3
(c) 3 only
(b) 3 only
(d) 2 and 3 only
(c) 2 and 4 only
(d) 1 and 4 only Solution 5. Correct Option: (a)
Option (a) is correct
Solution 4. Correct Option: (a)
Option (a) is correct • Currency devaluation
oslo is the intentional lowering
of a country's currency value against foreign
• Foreign Direct Investment (FDI) is an investment currencies.
by a company from one country into a company
from another country with the intention of • This strategy aims to boost exports by making
establishing a lasting interest. domestic goods and services cheaper abroad.

• FDI into India comes through three main routes: • However, devaluation can increase the cost of
imports and the domestic currency's foreign value,
• Automatic route FDI allows investment without potentially not improving the trade balance over
prior approval from the government or the Reserve time.
Bank of India (RBI).
6. If another global financial crisis happens in the
• Government route FDI requires prior approval
near future, which of the following actions/policies
from the government.
are most likely to give some immunity to India?
[2020]
• Foreign Currency Convertible Bonds (FCCBs) are
considered under FDI policy as they can be
(1) Not depending on short-term foreign
converted into equity shares.
borrowings

https://www.meritnest.com Page 14
Economy PYQs 3 of 5 External Sector, International Institutions

(2) Opening up to more foreign banks (b) It is a largely non-debt creating capital flow.
(c) It is the investment which involves
(3) Maintaining full capital account convertibility debt-servicing.
(d) It is the investment made by foreign institutional
Select the correct answer using the code given
investors in the Government securities.
below:
(a) 1 only [email protected]
8. Correct Option: (b)
(b) 1 and 2 only
Option (b) is correct
(c) 3 only
(d) 1, 2 and 3 • Refer to answer 2 for context on Foreign Direct
Investment (FDI) being non-debt creating.
Solution 6. Correct Option: (a)
• Foreign Portfolio Investment (FPI) involves
Option (a) is correct investments by individuals or institutional investors
from outside India in Indian listed companies.
• Foreign debt includes money borrowed from
foreign lenders by governments, corporations, or
households. 9. With reference to the international trade of
India at present, which of the following statements
• Short-term foreign borrowings are more volatile is/are correct? [2020]
and can lead to a liquidity crisis during global
financial turmoil. (1) India’s merchandise exports are less than its
merchandise imports.
• Minimizing reliance on short-term foreign
borrowings can provide some protection against (2) India’s imports of iron and steel, chemicals,
global crises. fertilisers and machinery have decreased in recent
years.

7. “Gold Tranche” (Reserve Tranche) refers to (3) India’s exports of services are more than its
[2020] imports of services.
(a) a loan system of the World Bank
(4) India suffers from an overall trade/current
(b) one of the operations of a Central Bank
account deficit.
(c) a credit system granted by WTO to its members
(d) a credit system granted by IMF to its members Select the correct answer using the code given
below:
Solution 7. Correct Option: (d) (a) 1 and 2 only
oslo
Option (d) is correct (b) 2 and 4 only
(c) 3 only
• The Gold Tranche or Reserve Tranche represents
(d) 1, 3 and 4 only
a portion of a member country's quota in the
International Monetary Fund (IMF) available in gold
or foreign currency. Solution 9. Correct Option: (d)
Option (d) is correct
• This tranche allows IMF members to access funds
without conditions, service fees, and consists of • As of August 2020, India's merchandise exports
25% of their quota, with the remaining 75% in were valued at USD 22.70 billion, while imports
domestic currencies for credit purposes. were USD 29.47 billion, indicating a trade deficit.

9654469135
8. With reference to Foreign Direct Investment in • There's been a decrease in imports of iron and
India, which one of the following is considered its steel but an increase in chemicals, fertilizers, and
major characteristic? [2020] machinery as per Economic Survey 2020.

(a) It is the investment through capital instruments • Service exports amounted to USD 84.47 billion
essentially in a listed company. compared to service imports of USD 49.56 billion

https://www.meritnest.com Page 15
Economy PYQs 3 of 5 External Sector, International Institutions

for April-August 2020-21. (c) Easing conditions relating to external


commercial borrowing.
• A trade deficit occurs when goods' exports are (d) Following an expansionary monetary policy.
less than imports, contributing to a current account
oslo
deficit due to the overall negative trade balance.
9654469135
Solution 11. Correct Option: (d)
Option (d) is correct
10. With reference to Trade-Related Investment
Measures (TRIMS), which of the following
• Expansionary monetary policy by the Reserve
statements is/are correct? [2020]
Bank of India (RBI) involves increasing money
supply, reducing interest rates, and boosting
(1) Quantitative restrictions on imports by foreign
aggregate demand.
investors are prohibited. [email protected]
• Lower domestic interest rates may lead to
(2) They apply to investment measures related to
currency outflows, potentially devaluing the Indian
trade in both goods and services.
Rupee.
(3) They are not concerned with the regulation of
foreign investment. 12. Which one of the following is not a sub-index
of the World Bank’s ‘Ease of Doing Business
Select the correct answer using the code given Index’? [2019]
below:
(a) Maintenance of law and order
(a) 1 and 2 only (b) Paying taxes
(b) 2 only (c) Registering property
(c) 1 and 3 only (d) Dealing with construction permits
(d) 1, 2 and 3
Solution 12. Correct Option: (a)
Solution 10. Correct Option: (d)
Option (a) is correct.
Option (d) is correct.
• Doing Business measures regulations affecting 11
• Under the Agreement on Trade-Related areas of a business's life, with ten of these areas
Investment Measures (TRIMS) of the World Trade included in the Doing Business 2019 ranking on the
Organization (WTO), members agreed not to apply Ease of Doing Business.
certain investment measures that discriminate
against foreign goods and restrict or distort trade. • These areas are: starting a business, dealing with
construction permits, getting electricity, registering
• This agreement, negotiated during the Uruguay property, getting credit, protecting minority
Round (1986-1994), applies only to measures investors, paying taxes, trading across borders,
affecting trade in goods and does not regulate enforcing contracts, and resolving insolvency.
foreign investment itself.
13. The Global Competitiveness Report is
• The TRIMS Agreement focuses on investment
published by the [2019]
measures that discriminate between imported and
exported products, upholding WTO principles of (a) International Monetary Fund
national treatment under GATT Article III and (b) United Nations Conference on Trade and
avoiding quantitative restrictions under Article XI. Development
(c) World Economic Forum
11. Which one of the following is not the most (d) World Bank
likely measure the Government/RBI takes to stop
the slide of Indian rupee? [2019]
Solution 13. Correct Option: (c)
(a) Curbing imports of non-essential goods and
Option (c) is correct.
promoting exports.
(b) Encouraging Indian borrowers to issue rupee • The Global Competitiveness Report (GCR) is
denominated Masala Bonds. published yearly by the World Economic Forum

https://www.meritnest.com Page 16
Economy PYQs 3 of 5 External Sector, International Institutions

(WEF), a Swiss nonprofit foundation established in Which of the statements given above is/are
1971, based in Geneva, Switzerland. correct?
9654469135
(a) 1 and 2 only
• The WEF is dedicated to improving the state of
the world by engaging business, political, academic, (b) 1 and 3 only
and other leaders of society to shape global, (c) 2 and 3 only
regional, and industry agendas. (d) 1, 2 and 3
[email protected]
• Major reports published by the WEF include the
Solution 15. Correct Option: (a)
Energy Transition Index, Global Competitiveness
Report, Global IT Report (in collaboration with Option (a) is correct.
INSEAD and Cornell University), Global Gender
Gap Report, Global Risk Report, and Global Travel • The Trade Facilitation Agreement (TFA) aims to
and Tourism Report. expedite global trade by reforming customs
procedures and reducing red tape.
14. India enacted the Geographical Indications of
• The TFA is the World Trade Organization’s
Goods (Registration and Protection) Act, 1999 in
(WTO) first-ever multilateral accord intended to
order to comply with the obligations to [2018]
simplify customs regulations for the cross-border
(a) ILO movement of goods, resulting from the WTO’s 9th
(b) IMF Bali (Indonesia) ministerial package of 2013.
(c) UNCTAD
• India ratified the TFA in April 2016, and the
(d) WTO agreement came into force in 2017, aiming to boost
global growth by reducing transaction costs for
Solution 14. Correct Option: (d) exports and imports.
Option (d) is correct.
16. With reference to the 'National Intellectual
• A geographical indication (GI) is a name, sign, or Property Rights Policy', consider the following
indication used on products that denotes a specific statements: [2017]
geographical location or origin.
(1) It reiterates India’s commitment to the Doha
• Geographical indications are protected under Development Agenda and the TRIPS Agreement.
Articles 1(2) and 10 of the Paris Convention for the
Protection of Industrial Property and Articles 22 to (2) Department of Industrial Policy and Promotion
24 of the Trade-Related Aspects of Intellectual is the nodal agency for regulating intellectual
Property Rights (TRIPS) Agreement, part of the property rights
osloin India.
Agreements concluding the Uruguay Round of
General Agreement on Tariffs and Trade (GATT) Which of the above statements is/are correct?
negotiations.
(a) 1 only
• India, as a World Trade Organization (WTO) (b) 2 only
member, enacted the Geographical Indications of (c) Both 1 and 2
Goods (Registration & Protection) Act, 1999, which (d) Neither 1 nor 2
came into effect from 15th September 2003.
Solution 16. Correct Option: (c)
15. Consider the following statements. [2017]
Option (c) is correct.
(1) India has ratified the Trade Facilitation
• The National Intellectual Property Rights (NIPR)
Agreement (TFA) of WTO.
Policy was approved by the Union Cabinet in 2016
to establish the future roadmap for intellectual
(2) TFA is a part of WTO’s Bali Ministerial Package
property in India.
of 2013.
• The Department of Industrial Policy & Promotion
(3) TFA came into force in January 2016.
(DIPP), under the Ministry of Commerce, has been

https://www.meritnest.com Page 17
Economy PYQs 3 of 5 External Sector, International Institutions

designated as the nodal department for Intellectual • SDRs, an international reserve asset, aim to
Property Rights (IPRs), tasked with coordinating, supplement IMF member countries' official
guiding, and overseeing the implementation and reserves.
development of IPRs in India.
• The SDR basket also includes the U.S. Dollar,
• The Cell for IPR Promotion & Management Euro, British Pound, and Japanese Yen, with
(CIPAM) has been established under DIPP to currencies needing to be freely usable, widely used,
implement the objectives of the National IPR Policy. and traded for inclusion.
[email protected]

• India’s IPR regime complies with the WTO’s


19. Which of the following best describes the term
Agreement on Trade-Related Aspects of
‘import cover’, sometimes seen in the news? [2016]
Intellectual Property Rights (TRIPS), reiterating
India’s commitment to the Doha Development (a) It is the ratio of value of imports to the Gross
Agenda and the TRIPS agreement. Domestic Product of a country.
(b) It is the total value of imports of a country in a
17. Which of the following gives 'Global Gender year.
Gap Index' ranking to the countries of the world? (c) It is the ratio between the value of exports and
[2017] that of imports between two countries.
(a) World Economic Forum (d) It is the number of months of imports that could
be paid for by a country’s international reserves.
(b) UN Human Rights Council
(c) UN Women
Solution 19. Correct Option: (d)
(d) World Health Organisation
Option (d) is correct
Solution 17. Correct Option: (a)
• Import cover is a metric for assessing a country's
Option (a) is correct. foreign exchange reserve adequacy in covering
import costs.
The Global Gender Gap Index, published by the
World Economy Forum, annually benchmarks the • It reflects the number of months a country can
current state and evolution of gender parity across continue to import goods using its existing foreign
four key dimensions (Economic Participation and exchange reserves.
Opportunity, Educational Attainment, Health and
Survival, and Political Empowerment). It is the • India currently has an import cover of
longest-standing index tracking the progress of approximately 12 months, aiding in Balance of
numerous countries’ efforts towards closing these Payment (BoP) crisis prevention and facilitating
gaps over time since its inception in 2006. early intervention measures.

oslo
18. Recently, which one of the following 20. ‘Global Financial Stability Report’ is prepared
currencies has been proposed to be added to the by the [2016]
basket of IMF’s SDR? [2016]
(a) European Central Bank
(a) Rouble (b) International Monetary Fund
(b) Rand (c) International Bank for Reconstruction and
(c) Indian Rupee Development
(d) Renminbi (d) Organisation for Economic Cooperation and
Development
Solution 18. Correct Option: (d)
Solution 20. Correct Option: (b)
Option (d) is correct
9654469135
Option (b) is correct.
• The IMF's Executive Board included the Chinese
Renminbi (Yuan) in the Special Drawing Right (SDR) • The International Monetary Fund (IMF) was
currency basket from October 1, 2016. initiated in 1944 at the Bretton Woods Conference
and began operations on 27th December 1945 with
189 member countries.

https://www.meritnest.com Page 18
Economy PYQs 3 of 5 External Sector, International Institutions

(b) 2 only
• Its headquarters is in Washington, D.C., focusing (c) Both 1 and 2
on fostering global monetary cooperation, securing
(d) Neither 1 nor 2
financial stability, facilitating international trade,
oslo
and promoting employment and economic growth
9654469135
worldwide. Solution 22. Correct Option: (c)
Option (c) is correct.
• The IMF publishes reports including the World
Economic Outlook (WEO), Global Financial Stability • The International Monetary and Financial
Report (GFSR), and Fiscal Monitor (FM). Committee (IMFC) advises and reports to the IMF
Board of Governors on managing the international
21. In the context of which of the following do you monetary and financial system.
[email protected]
sometimes find the terms ‘amber box, blue box
and green box’ in the news? [2016] • While the IMFC has no formal decision-making
powers, it plays a key role in providing strategic
(a) WTO affairs direction to the IMF's work and policies.
(b) SAARC affairs
(c) UNFCCC affairs • International institutions, including the World
Bank, participate as observers in IMFC meetings.
(d) India-EU negotiations on FTA

Solution 21. Correct Option: (a) 23. Convertibility of rupee implies: [2015]

Option (a) is correct. (a) Being able to convert rupee notes into gold.
(b) Allowing the value of rupee to be fixed by
• In World Trade Organization (WTO) terminology, market forces.
subsidies are categorized by "Boxes" with traffic (c) Freely permitting the conversion of rupee to
lights colors: green (permitted), amber (to be other currencies and vice versa.
reduced), and red (forbidden).
(d) Developing an international market for
currencies in India.
• The 'amber box' includes all domestic support
measures considered to distort production and
trade, with some exceptions. Solution 23. Correct Option: (c)
Option (c) is correct
• The 'green box' subsidies must not distort trade,
or at most cause minimal distortion, to qualify. • Rupee convertibility refers to the ability to
convert the Indian rupee into any other foreign
• The 'blue box' is an exemption from the general currency without restrictions regarding the
rule that all production-linked subsidies must be purpose of the currency exchange.
reduced or kept within minimal ("de minimis")
levels.
24. ‘Basel III Accord’ or simply ‘Basel III’, often
seen in the news, seeks to [2015]
22. With reference to the International Monetary
and Financial Committee (IMFC), consider the (a) develop national strategies for the conservation
following statements: [2016] and sustainable use of biological diversity.
(b) improve banking sector’s ability to deal with
(1) IMFC discusses matters of concern affecting the financial and economic stress and improve risk
global economy, and advises the International management.
Monetary Fund (IMF) on the direction of its work. (c) reduce the greenhouse gas emissions but places
a heavier burden on developed countries.
(2) The World Bank participates as observer in
(d) transfer technology from developed countries to
IMFC’s meetings.
poor countries to enable them to replace the use of
chlorofluorocarbons in refrigeration with harmless
Which of the statements given above is/are
chemicals.
correct?
(a) 1 only Solution 24. Correct Option: (b)

https://www.meritnest.com Page 19
Economy PYQs 3 of 5 External Sector, International Institutions

Option (b) is correct. (b) United Nations Framework Conference on


Climate Change
• The Basel III Accord is a global, voluntary (c) World Trade Organization
regulatory framework on bank capital adequacy, (d) United Nations Environment Programme
stress testing, and market liquidity risk, aimed at
strengthening the international banking system.
Solution 26. Correct Option: (c)
• Agreed upon by the Basel Committee on Banking Option (c) is correct.
Supervision members in 2010–11, these guidelines
were a response to the 2008 financial crisis. • The Agreement on Agriculture (AoA) is an
international treaty of the World Trade
• Basel III focuses on enhancing the resilience of the Organization (WTO).
banking system by emphasizing capital, leverage,
funding, and liquidity. • It was negotiated during the Uruguay Round of
the General Agreement on Tariffs and Trade
(GATT) and came into force with the establishment
25. Which one of the following issues the “Global
of the WTO on January 1, 1995.
Economic Prospects” report periodically? [2015]
[email protected]
(a) The Asian Development Bank
27. Which of the following organizations brings
(b) The European Bank for Reconstruction and out the publication known as 'World Economic
Development Outlook'? [2014]
(c) The US Federal Reserve Bank
(a) The International Monetary Fund
(d) The World Bank
(b) The United Nations Development Programme
(c) The World Economic Forum
Solution 25. Correct Option: (d)
(d) The World Bank
Option (d) is correct.
Solution 27. Correct Option: (a)
• The International Bank for Reconstruction and
Development (IBRD), known as the World Bank, Option (a) is correct.
was founded in 1944 and has grown into a group of
five development institutions. • The World Economic Outlook (WEO) is a report
by the International Monetary Fund (IMF) that
• The World Bank Group includes the IBRD, analyzes key aspects of the IMF’s surveillance of
International Development Association (IDA), economic developments and policies in its member
International Finance Corporation (IFC), countries, and projects developments in the global
Multilateral Guarantee Agency (MIGA), and financial markets and economic systems.
International Centre for the Settlement of
Investment Disputes (ICSID).
28. The balance of payments of a country is a
systematic record of [2013]
• The IBRD offers loans, credits, and grants; the IDA
9654469135
provides low or no-interest loans to low-income (a) All import and export transactions of a country
countries; the IFC supports private sector during a given period of time, normally a year.
development; MIGA insures against political risks; (b) Goods exported from a country during a year.
and ICSID settles investment disputes.
(c) Economic transaction between the governments
of one country to another.
• The World Bank periodically releases the Global
Economic Prospects report. (d) Capital movements from one country to
another.

26. The terms ‘Agreement on Agriculture’,


Solution 28. Correct Option: (a)
‘Agreement on the Application of Sanitary and
Phytosanitary Measures’ and 'Peace Clause' Option (a) is correct
appear in the news frequently in the context of the
affairs of the [2015] oslo • The Balance of Payments (BOP) is a
comprehensive record of all economic transactions
(a) Food and Agriculture Organization

https://www.meritnest.com Page 20
Economy PYQs 3 of 5 External Sector, International Institutions

between residents of a country and the rest of the rates, interest rates, economic growth, foreign
world over a specified period, typically quarterly or investment inflows, and geopolitical conditions.
annually.
• The World Bank provides financial and technical
assistance to developing countries but does not
29. Which one of the following groups of items is
directly influence the price of currencies in the
included in India’s Foreign Exchange Reserves?
international market.
[2013]
(a) Foreign-currency assets, Special Drawing Rights • Income levels can affect consumer spending,
(SDRs) and loans from foreign countries. impacting the demand for imported goods and
(b) Foreign-currency assets, gold holdings of the thereby affecting the local currency's strength.
RBI and SDRs. oslo
• The balance of payments, including the trade
(c) Foreign currency assets, loans from the World
balance and capital flows, directly impacts the value
Bank and SDRs.
of a country's currency.
(d) Foreign currency assets, gold holdings of the RBI
and loans from the World Bank. • Trade surpluses, where a country exports more
than it imports, lead to an influx of foreign currency,
Solution 29. Correct Option: (b) strengthening the local currency.
Option (b) is correct
• Differences in interest rates between countries
can affect currency values, with higher rates
• Forex reserves include assets held by a central
attracting foreign capital, increasing demand for the
bank in foreign currencies, gold reserves, Special
local currency, and potentially strengthening it.
Drawing Rights (SDRs) with the International
Monetary Fund (IMF), and other forms of reserve
assets. These reserves serve as a financial buffer 31. Consider the following actions which the
and can influence a country's currency value. Government can take: [2011]

(1) Devaluing the domestic currency.


30. Consider the following statements: [2012]
(2) Reduction in the export subsidy.
The price of any currency in international market is
decided by the: [email protected]
(3) Adopting suitable policies which attract greater
FDI and more funds from FIIs.
(1) World Bank
Which of the above action/actions can help in
(2) demand for goods/services provided by the
reducing the current account deficit?
country concerned
(a) 1 and 2 only
(3) stability of the government of the concerned (b) 2 and 3 only
country
(c) 3 only
9654469135
(4) economic potential of the country in question (d) 1 and 3 only

Which of the statements given above are correct? Solution 31. Correct Option: (a)
(a) 1, 2, 3 and 4 Option (a) is correct
(b) 2 and 3 only
• The Current Account Deficit (CAD) measures a
(c) 3 and 4 only
country's trade where imports of goods and
(d) 1 and 4 only services exceed exports.

Solution 30. Correct Option: (b) • The CAD includes net income, such as interest
and dividends, and transfers, like foreign aid,
Option (b) is correct
though these are a smaller portion of the total
current account.
• Currency value is influenced by a variety of
economic factors including trade balances, inflation

https://www.meritnest.com Page 21
Economy PYQs 3 of 5 External Sector, International Institutions

• The CAD formula is: Trade gap + Net current (1) The teaser loans are considered to be an aspect
transfers + Net income abroad. of sub-prime lending and banks may be exposed to
9654469135
the risk of defaulters in future.
• Strategies to reduce CAD involve:
(2) In India, the teaser loans are mostly given to
Devaluing the currency to make exports cheaper
inexperienced entrepreneurs to set up
and imports more expensive.
manufacturing or export units.
Reducing domestic consumption and import
[email protected]
spending through tighter fiscal policy and increased Which of the statements given above is/are
taxes. correct?
Implementing supply-side policies to boost (a) 1 only
domestic industry and exports. (b) 2 only
Formulating policies to attract more Foreign Direct (c) Both 1 and 2
Investment (FDI). (d) Neither 1 nor 2

32. In the context of Indian economy, consider the Solution 33. Correct Option: (a)
following statements: [2011]
Option (a) is correct.
(1) The growth rate of GDP has steadily increased in
• A teaser loan offers a lower interest rate in the
the last five years.
initial years, followed by market-linked rates
thereafter.
(2) The growth rate in per capita income has
steadily increased in the last five years.
• For example, a home loan for 30 years at an
interest rate of 8% might have a teaser rate of 6%
Which of the statements given above is/are
for the first three years, before switching to 8%.
correct?
(a) 1 only • Common examples of teaser loans include credit
(b) 2 only cards with low introductory offers and
adjustable-rate mortgages.
(c) Both 1 and 2
(d) Neither 1 nor 2
34. The SEZ Act, 2005 which came into effect in
February 2006 has certain objectives. In this
Solution 32. Correct Option: (d)
context, consider the following: [2010]
Option (d) is correct
(1) Development
oslo of infrastructure facilities.
• Gross Domestic Product (GDP) is the total
monetary value of all finished goods and services (2) Promotion of investment from foreign sources.
produced within a country in a specific period.
(3) Promotion of exports of services only.
• GDP is used to estimate the size of an economy
and its growth rate, calculated through Which of the above are the objectives of this Act?
expenditures, production, or incomes.
(a) 1 and 2 only
• It can be adjusted for inflation and population for (b) 3 only
more detailed analysis. (c) 2 and 3 only
(d) 1, 2 and 3
• Per capita income is the average income per
person in a specific area in a given year, calculated
Solution 34. Correct Option: (a)
by dividing the total income by the population.
Option (a) is correct
33. Why is the offering of "teaser loans" by
• Special Economic Zones (SEZs) are designated
commercial banks a cause of economic concern?
areas with different business and trade laws from
[2011]
the rest of the country, aimed at enhancing trade

https://www.meritnest.com Page 22
Economy PYQs 3 of 5 External Sector, International Institutions

balance, employment, investment, and (3) A Trade Mark is assigned to the manufactured
infrastructure. goods whereas the Geographical Indication is
assigned to the agricultural goods/products and
• Objectives of India's SEZ policy include economic handicrafts only.
activity creation, export increase, domestic and
foreign investment promotion, employment Which of the statements given above is/ are
generation, and infrastructure development. correct?
(a) 1 only
• A 2019 committee recommended converting
SEZs into employment and economic enclaves with (b) 1 and 2 only
efficient infrastructure. (c) 2 and 3 only
(d) 1, 2 and 3 oslo
• The Finance Minister announced a new legislation
in the 2021-22 Budget speech to replace the SEZ
Solution 36. Correct Option: (b)
Act, encouraging states to become development
hubs. Option (b) is correct.

• A trademark is a type of intellectual property


35. Which of the following is/are treated as
consisting of a recognizable sign, design, or
artificial currency? [2010]
expression that identifies and distinguishes
(a) ADR products or services of a particular source from
(b) GDR others.
(c) SDR
• Trademark ownership can belong to individuals,
(d) Both ADR and SDR business organizations, or any legal entities.

Solution 35. Correct Option: (c) • Trademarks are used on packages, labels,
vouchers, or directly on products, often consisting
Option (c) is correct
of arbitrary signs that may be used by the owner or
authorized persons.
• The Special Drawing Right (SDR) is an
international reserve asset created by the IMF in
• Trademarks can be allocated or licensed globally,
1969 to supplement member countries' official
as they are linked to specific companies rather than
[email protected]
reserves.
locations.
• The SDR's value is based on a basket of five
• A geographical indication (GI) signifies products
currencies: U.S. dollar, euro, Chinese renminbi,
with a specific geographical origin, possessing
Japanese yen, and British pound sterling.
qualities or reputation stemming from that origin.
• It represents a potential claim on the freely usable
• GIs are used for agricultural products, foodstuffs,
currencies of IMF members, allowing for exchange
wine and spirit drinks, handicrafts, and industrial
into these currencies.
products, and can be used by anyone in the origin
9654469135
area who produces the goods according to specified
36. In order to comply with TRIPS Agreement, standards.
India enacted the Geographical Indications of
Goods (Registration & Protection) Act, 1999. The • Unlike trademarks, a GI cannot be allocated or
difference/differences between a "Trade Mark" licensed to individuals outside the origin place or
and a Geographical Indication is/are: [2010] not part of the authorized producer group.

(1) A Trade Mark is an individual or a company’s


37. Consider the following countries: [2010]
right whereas a Geographical Indication is a
community’s right.
(1) Brazil
(2) A Trade Mark can be licensed whereas a
(2) Mexico
Geographical Indication cannot be licensed.
(3) South Africa

https://www.meritnest.com Page 23
Economy PYQs 3 of 5 External Sector, International Institutions

According to UNCTAD, which of the above is/are


• IDA focuses on reducing poverty by providing
categorized as "Emerging Economies"?
zero to low-interest loans ("credits") and grants for
(a) 1 only programs aimed at strengthening economic growth,
(b) 1 and 3 only reducing inequalities, and improving living
conditions.
(c) 2 and 3 only
(d) 1, 2 and 3
39. As regards the use of international food safety
standards as reference point for the dispute
Solution 37. Correct Option: (d)
settlements, which one of the following does WTO
Option (d) is correct. collaborate with? [2010]
(a) Codex Alimentarius Commission
• The United Nations Conference on Trade and
Development (UNCTAD) was established in 1964 (b) International Federation of Standards Users
as an intergovernmental organization to support (c) International Organization for Standardization
developing states in world trade. (d) World Standards Cooperation

• UNCTAD is part of the United Nations Secretariat


[email protected] Solution 39. Correct Option: (a)
dealing with trade, investment, and development
issues. Option (a) is correct.

• An emerging market is a market with some • The Codex Alimentarius Commission (CAC) is an
characteristics of a developed market but does not international food standards body established in
meet all its standards, potentially becoming a May 1963 by the Food and Agriculture
developed market in the future or having been one Organization (FAO) and the World Health
in the past. Organization (WHO).

• The term "frontier market" refers to developing • Its aim is to protect consumer health and ensure
countries with smaller, riskier, or more illiquid fair practices in the food trade.
capital markets than "emerging" markets.
• The Agreement on the Application of Sanitary and
• The 10 largest emerging and developing Phytosanitary Measures (SPS) of the World Trade
economies by either nominal GDP or PPP-adjusted Organization (WTO) recognizes Codex standards,
GDP include four of the five BRICS countries guidelines, and recommendations as standards for
(Brazil, Russia, India, and China), along with international trade and dispute settlement.
Indonesia, Iran, South Korea, Mexico, Saudi Arabia,
Taiwan, and Turkey. • India became a member of the Codex
Alimentarius in 1964.
38. The International Development Association, a
lending agency, is administered by the [2010] 40. In the context of the affairs of which of the
following is the phrase "Special Safeguard
(a) International Bank for Reconstruction and 9654469135
Mechanisms" mentioned in the news frequently?
Development
[2010]
(b) International Fund for Agricultural
Development (a) United Nations Environment Programme
(c) United Nations Development Programme (b) World Trade Organization
(d) United Nations Industrial Development (c) ASEAN - India Free Trade Agreement
Organization (d) G-20 Summits

Solution 38. Correct Option: (a) Solution 40. Correct Option: (b)
Option (a) is correct. Option (b) is correct.

• The International Development Association (IDA) • The Special Safeguard Mechanism in the Doha
oslo
is part of the World Bank, established in 1960 to Round of the WTO allows developing countries to
assist the world's poorest countries. temporarily raise tariffs to deal with import surges

https://www.meritnest.com Page 24
Economy PYQs 3 of 5 External Sector, International Institutions

or price falls, associated with the World Trade countries.


Organization (WTO).
9654469135
43. Who among the following is the founder of
41. Other than Venezuela, which one among the World Economic Forum? [2009]
following from South America is a member of
(a) Klaus Schwab
OPEC? [2009]
(b) John Kenneth Galbraith
(a) Argentina
(c) Robert Zoellick
[email protected]
(b) Brazil
(d) Paul Krugman
(c) Ecuador
(d) Bolivia Solution 43. Correct Option: (a)
Option (a) is correct.
Solution 41. Correct Option: (c)
Option (c) is correct. • The World Economic Forum (WEF) was founded
on 24 January 1971 by Klaus Schwab, a German
• The Organization of the Petroleum Exporting engineer and economist.
Countries (OPEC) is an intergovernmental
organization or cartel of 13 countries, founded on
44. With reference to the United Nations, consider
14 September 1960 in Baghdad by five members
the following statements: [2009]
(Iran, Iraq, Kuwait, Saudi Arabia, and Venezuela).
(1) The Economic and Social Council (ECOSOC) of
• Since 1965, it has been headquartered in Vienna,
UN consists of 24 member States.
Austria.
(2) Member states are elected by a 2/3rd majority
• As of September 2018, the 13 member countries
of the General Assembly for a 3-year term.
accounted for an estimated 44 percent of global oil
production and 81.5 percent of the world's
Which of the above statements is/are correct?
"proven" oil reserves, significantly influencing
global oil prices. (a) 1 only
(b) 2 only
• Member countries include Qatar (1961),
(c) Both 1 and 2
Indonesia (1962), Libya (1962), the United Arab
Emirates (1967), Algeria (1969), Nigeria (1971), (d) Neither 1 nor 2
Ecuador (1973), Gabon (1975), Angola (2007),
Equatorial Guinea (2017), and Congo (2018). Solution 44. Correct Option: (b)
Option (b) isoslo
correct.
42. NAMA-11 group of countries frequently
appears in the news in the context of the affairs of • The Economic and Social Council (ECOSOC) is
which one of the following? [2009] central to the United Nations system for advancing
sustainable development across economic, social,
(a) Nuclear Suppliers Group
and environmental dimensions.
(b) World Bank
(c) World Economic Forum • It serves as a platform for debate, innovative
(d) W.T.O. thinking, consensus-building, and coordination to
achieve internationally agreed goals and follow-up
to major UN conferences and summits.
Solution 42. Correct Option: (d)
Option (d) is correct. • Established by the UN Charter in 1945 as one of
the six main organs of the United Nations.
• The Non-Agricultural Market Access (NAMA)
negotiations of the World Trade Organization are • Collen Vixen Kelapile was elected as the
based on the Doha Declaration of 2001. seventy-seventh President of ECOSOC on 23 July
2021.
• They call for reductions or eliminations in tariffs,
especially on goods of interest to developing

https://www.meritnest.com Page 25
Economy PYQs 3 of 5 External Sector, International Institutions

• ECOSOC has 54 member states elected annually


by the General Assembly for overlapping Which of the above pairs is/are correctly matched?
9654469135
three-year terms, with a two-thirds majority
(a) 1 only
required for election.
(b) 2 only
• Membership ensures equitable geographic (c) Both 1 and 2
rotation, allocated among the UN regional groups: (d) Neither 1 nor 2
14 seats for the African Group, 11 for the
[email protected]
Asia-Pacific group, 6 for the Eastern European
Solution 46. Correct Option: (b)
Group, 10 for the Latin American and Caribbean
Group, and 13 for the Western European and Option (b) is correct.
Others Group.
• The Asian Development Bank (ADB) was
• Over 1,600 non-governmental organizations have established on 19 December 1966 and is
consultative status with ECOSOC to participate in headquartered in the Ortigas Center,
the UN's work. Mandaluyong, Metro Manila, Philippines.

• The Asia-Pacific Economic Cooperation (APEC) is


45. Consider the following pairs:
an inter-governmental forum for 21 Pacific Rim
member economies that promotes free trade in the
Large Bank - Country of origin [2009]
Asia-Pacific region, started in 1989 with its
headquarters in Singapore.
(1) ABN Amro Bank - USA
(2) Barclays Bank - UK • The Association of Southeast Asian Nations
(ASEAN) was established on 8 August 1967 in
(3) Kookmin Bank - Japan
Bangkok, Thailand, by the signing of the ASEAN
Declaration (Bangkok Declaration) by Indonesia,
Which of the above pairs is/are correctly matched?
Malaysia, Philippines, Singapore, and Thailand, and
(a) 1 only is headquartered in Jakarta, Indonesia.
(b) 2 only
(c) 1 and 2 47. India is a member of which of the following?
(d) 2 and 3 [2008]

(1) Asian Development Bank


Solution 45. Correct Option: (b)
Option (b) is correct. (2) Asia-Pacific Economic Cooperation
oslo
• ABN AMRO Bank N.V. is headquartered in (3) Colombo Plan
Amsterdam, Netherlands.
(4) Organization for Economic Cooperation and
• Barclays Bank is a British multinational universal Development (OECD)
bank, headquartered in London, England.
Select the correct answer using the codes given
• Kookmin Bank (KB Kookmin Bank) is one of the below:
four largest banks in South Korea by asset value as
(a) 1 and 3 only
of the end of March 2014.
(b) 2 and 4 only
(c) 1, 2 and 3 only
46. Consider the following pairs:
(d) 1, 2, 3 and 4
Organisation - Location of Headquarters [2009]
Solution 47. Correct Option: (a)
(1) Asian Development Bank - Tokyo
Option (a) is correct.
(2) Asia-Pacific Economic Cooperation - Singapore
• As of 31 December 2020, India holds 6.317%
(3) Association of South East Asian Nations -
shares in the Asian Development Bank (ADB).
Bangkok

https://www.meritnest.com Page 26
Economy PYQs 3 of 5 External Sector, International Institutions

• India is not a member of the Asia-Pacific


Solution 49. Correct Option: (d)
Economic Cooperation (APEC).
9654469135
Option (d) is correct.
• The Colombo Plan for Cooperative, Economic and
Social Development in Asia and the Pacific was • The International Monetary Fund (IMF) was
initiated in January 1950 and launched on 1 July established in July 1944 at the Bretton Woods
1951, focusing on the economic and social Conference, promoting international financial
advancement of South and Southeast Asia. stability, monetary cooperation, international
[email protected]
trade, sustainable economic growth, and poverty
• It started with seven Commonwealth nations and reduction.
has expanded to 27 intergovernmental member
countries, including non-Commonwealth countries. • Raghuram Rajan served as the Chief Economist of
the IMF from September 2003 to January 2007,
• The Organisation for Economic Co-operation and holding a professorship at the University of Chicago
Development (OECD) is an intergovernmental at the time of his appointment.
economic organization with 38 member countries,
founded in 1961 to stimulate economic progress
50. Which one of the following is the correct
and world trade, not including India as a member.
statement? [2006]
• The OECD has established agencies such as the Service tax is a/an:
OECD Development Centre, International Energy
(a) Direct tax levied by the Central Government
Agency (IEA), and Financial Action Task Force on
Money Laundering. (b) Indirect tax levied by the Central Government
(c) Direct tax levied by the State Government
• India is not a member of the Organization of the (d) Indirect tax levied by the State Government
Petroleum Exporting Countries (OPEC).
Solution 50. Correct Option: (b)
48. Tarapore Committee was associated with
Option (b) is correct
which one of the following? [2007]
(a) Special Economic Zones • Service tax is an indirect tax collected by the
(b) Fuller capital account convertibility service provider from the service receiver and
remitted to the Government of India.
(c) Foreign exchange reserves
(d) Effect of oil-prices on the Indian economy
51. Consider the following statements: [2006]
Solution 48. Correct Option: (b) oslo
(1) In India, during the financial year 2004 – 2005,
Option (b) is correct an increase of below 10% over the value of exports
(in rupee terms) in the financial year 2003 – 2004
• Dr. Y.V. Reddy, former Governor of the Reserve was reported.
Bank of India (RBI), formed a committee in 2006 to
outline the path towards Full Capital Account (2) According to the WTO, India’s share in the world
Convertibility. merchandise exports crossed 2% in the year 2005.

• Shri S.S. Tarapore was appointed as the Chairman Which of the statement(s) given above is/are
of this committee. correct?
(a) 1 only
49. Who among the following served as the Chief (b) 2 only
Economist of the International Monetary Fund?
(c) Both 1 and 2
[2007]
(d) Neither 1 nor 2
(a) Ashok Lahiri
(b) Sumantra Ghoshal Solution 51. Correct Option: (d)
(c) Saumitra Chaudhuri
Option (d) is correct
(d) Raghuram Rajan

https://www.meritnest.com Page 27
Economy PYQs 3 of 5 External Sector, International Institutions

• In 2004-05, India's export growth was 24.9%, (d) 3 only


surpassing the government's annual target of 16%
and exceeding the growth rate of 2003-04.
Solution 53. Correct Option: (c)
• From 2002 to 2007, India's merchandise exports Option (c) is correct.
experienced robust growth, averaging 23.3%, with
the highest annual growth of 30.8% in 2004-05, the • India has approximately 154,761 Post Offices
most significant in over three decades. [email protected]
various services.

• Consequently, India's share in global merchandise • Bharat Sanchar Nigam Limited (BSNL) is a
exports increased to 1.0% in 2006 from 0.7% in government-owned telecommunication service
2000. provider headquartered in New Delhi, under the
Department of Telecommunications, Ministry of
Communications, established on 1 October 2000.
52. Which one of the following companies has
started a rural marketing network called
• The Telecom Regulatory Authority of India (TRAI)
‘e-chaupals’? [2005]
is the regulatory body for telecommunications in
(a) ITC India, established under the Telecom Regulatory
(b) Dabur Authority of India Act, 1997.
(c) Proctor & Gamble
(d) Hindustan Lever 54. Assertion (A): For the first time, India had no
trade deficit in the year 2002- 2003.
Solution 52. Correct Option: (a)
Reason (R): For the first time, India’s exports
Option (a) is correct crossed worth $50 billion in the year 2002-03.
[2004]
• E-Choupal is an initiative by ITC Limited to
(a) Both A and R are individually true and R is the
connect directly with rural farmers through the
correct explanation of A
internet for procuring agricultural and aquaculture
products such as soybeans, wheat, coffee, and (b) Both A and R are individually true but R is not
prawns. the correct explanation of A
(c) A is true but R is false
• This program addresses challenges in Indian (d) A is false but R is true
agriculture, marked by fragmented farming, poor
infrastructure, and intermediaries, by installing
Solution 54. Correct Option: (d)
computers with internet access in rural areas to
offer farmers current marketing and agricultural Option (d) is correct
information.
oslo
• India's share in global merchandise exports
increased to 0.8% in 2002 from 0.4% in 1992-93
53. Consider the following statements: [2005]
and 0.7% in 2001.
(1) The number of post offices in India is in excess of
• In 2002-03, India's merchandise goods exports
1.5 lakhs.
were valued at US $51.7 billion, surpassing the US
$50 billion milestone.
(2) Bharat Sanchar Nigam Limited (BSNL) was
formed in the year 1997.
55. Consider the following statements: India
(3) Telecom Regulatory Authority of India (TRAI) continues to be dependent on imports to meet the
was established in the year 2000. requirement of oilseeds in the country because:
[2004]
9654469135
Which of the statements given above is/are
correct? (1) Farmers prefer to grow food grains with highly
remunerative support prices.
(a) 1, 2 and 3
(b) 1 and 2 (2) Most of the cultivation of oilseed crops
(c) 1 only continues to be dependent on rainfall.

https://www.meritnest.com Page 28
Economy PYQs 3 of 5 External Sector, International Institutions

Codes:
(3) Oils from the seeds of tree origin and rice bran
have9654469135
remained unexploited. ABCD
(a) 2 3 4 1
(4) It is far cheaper to import oilseeds than to
cultivate the oilseed crops. (b) 4 1 2 3
(c) 2 1 4 3
Which of the statements given above is/are (d) 4 3 2 1
[email protected]
correct?
(a) 1 and 2 Solution 56. Correct Option: (b)
(b) 1, 2 and 3 Option (b) is correct.
(c) 3 and 4
(d) 1, 2, 3 and 4 • The United Nations Development Programme
(UNDP) is headquartered in New York, USA.
Solution 55. Correct Option: (a)
• The United Nations Environment Programme
Option (a) is correct (UNEP) is headquartered in Nairobi, Kenya.

• India's demand for edible oil significantly exceeds • The United Nations Industrial Development
domestic production, leading to substantial Organization (UNIDO) is headquartered in Vienna,
imports, accounting for about 25% of India's total Austria.
import bill.
• The Universal Postal Union (UPU) is
• The inclination of Indian farmers towards headquartered in Bern, Switzerland.
cultivating commodities with higher support prices
is due to the high cost of cultivation and price
57. Consider the following given statements:
volatility in the global market.
[2003]
• The yield of various oilseed crops in India is below
(1) The maximum limit of shareholding of Indian
the global average and considerably lower than in
promoters in private sector banks in India is 49
other leading oilseed-producing nations, with 75%
percent of the paid-up capital.
of the oilseed production area being rainfed.
(2) Foreign Direct Investment up-to 49 percent
56. Match List-I (Agency) with List-II from all sources is permitted in private sector banks
(Headquarters) and select the correct answer in India under the automatic route.
using the codes given below the Lists: [2004] oslo
Which of these statements is/are correct?
List-I (Agency)
(a) 1 only
(A) United Nations Development Programme (b) 2 only
(UNDP)
(c) Both 1 and 2
(B) United Nations Environment Programme (d) Neither 1 nor 2
(UNEP)
(C) United Nations Industrial Development Solution 57. Correct Option: (b)
Organization (UNIDO)
Option (b) is correct
(D) Universal Postal Union (UPU)
• For Civil-aviation (Greenfield projects and
List-II (Headquarters) Existing projects), the Foreign Direct Investment
(FDI) cap/equity is 100% through the Automatic
1. Nairobi
entry route.
2. Vienna
• Asset Reconstruction companies have an FDI cap
3. Berne
of 49% through the Foreign Investment Promotion
4. New York Board (FIPB) route.

https://www.meritnest.com Page 29
Economy PYQs 3 of 5 External Sector, International Institutions

(b) RIL is the largest private sector company in India


• The Banking (private) sector allows up to 74% FDI
and Foreign Institutional Investor (FII) investment, (c) MTNL is listed on NYSE
9654469135
with the condition that FII does not exceed 49%, (d) BSNL is the first telecom service organization in
accessible through the Automatic route. India to launch a nationwide cellular service at one
time
• Non-Banking Financial Companies (NBFCs)
involved in underwriting, portfolio management Solution 59. Correct Option: (a)
services, investment advisory services, [email protected]
financial
consultancy, stock broking, asset management, Option (a) is correct
venture capital, custodian services, factoring,
leasing and finance, housing finance, forex broking, • Indian Petrochemicals Corporation Limited (IPCL)
etc., have a 100% FDI cap through the Automatic was established on 22 March 1969 by the
route. Government of India to promote the development
of the petrochemical industry.

58. Assertion (A): The new EXIM policy is liberal, • Reliance Industries Limited (RIL) is the largest
market-oriented and favours global trade. petrochemical producer in India and among the top
ten globally.
Reason (R): GATT has played a significant role in the
liberalisation of the economy. [2003] • Mahanagar Telephone Nigam Limited (MTNL) is a
(a) Both A and R are individually true and R is the wholly-owned subsidiary of Bharat Sanchar Nigam
correct explanation of A Limited (BSNL), headquartered in New Delhi.
(b) Both A and R are individually true but R is not
the correct explanation of A 60. The government holding in BPCL is: [2003]
(c) A is true but R is false (a) more than 70%
(d) A is false but R is true (b) between 60% and 70%
(c) between 50% and 60%
Solution 58. Correct Option: (c)
(d) less than 50%
Option (c) is correct
Solution 60. Correct Option: (b)
• The aim of trade policy reforms has focused on
liberalization, openness, transparency, and Option (b) is correct
globalization, emphasizing export promotion and
moving away from quantitative restrictions. • Bharat Petroleum Corporation Limited (BPCL) is a
government-owned oil and gas corporation,
• Changes in the Export-Import (EXIM) policy have operating under
oslo the Ministry of Petroleum and

strengthened the export production base, removed Natural Gas, headquartered in Mumbai.
procedural hurdles, facilitated input availability,
focused on quality and technological upgradation, • BPCL operates two major refineries in Kochi and
and improved competitiveness. Mumbai and is India’s second-largest downstream
oil corporation.
• The General Agreement on Tariffs and Trade
(GATT) deals with international trade in goods. • It is classified as a Maharatna public sector
undertaking in India.
• The Goods Council, responsible for the workings
of GATT, consists of representatives from all World • The government currently holds 53.3% of BPCL's
Trade Organization (WTO) member countries, shares, a reduction from the earlier figure of around
currently chaired by Ambassador Lundeg 66%.
Purevsuren (Mongolia).
61. India’s external debt increased from US
59. With reference to India, which one of the $98,158 million as at the end of March 2000 to US
following statements is NOT correct? [2003] $100,225 million as at the end of March 2001 due
to increase in: [2002]
(a) IPCL is India’s largest petrochemical company
(a) multilateral and bilateral debt

https://www.meritnest.com Page 30
Economy PYQs 3 of 5 External Sector, International Institutions

(b) rupee debt (3) it acts just like any other international currency.
(c) commercial borrowings and NRI deposits
Which of these statements are correct?
(d) borrowing from International Monetary Fund
(a) 1 and 2
Solution 61. Correct Option: (a) (b) 1 and 3
Option (a) is correct (c) 2 and 3
(d) 1, 2 and 3
• India's external debt comprises debts owed to
foreign creditors by the Union government, state Solution 63. Correct Option: (d)
governments, corporations, or Indian citizens.
Option (d) is correct oslo
• The debt includes obligations to private
commercial banks, foreign governments, and • Convertibility refers to the ease of converting a
international financial institutions like the IMF and country's currency into gold or another currency
the World Bank. through global exchanges.

• In March 2001, India's external debt increased • In the 1992-93 budget, the Government of India
significantly due to enhanced multilateral and introduced partial convertibility of the Indian
bilateral debt. Rupee.

• Full convertibility of the Rupee implies


62. Global capital-flows to developing countries unrestricted conversion into foreign currency for all
increased significantly during the nineties. In view transactions, both current account and capital
of the East Asian financial crisis and Latin account.
American experience, which type of inflow is good
for the host country? [2002] • Full convertibility would allow the Rupee's
(a) Commercial loans exchange rate to be determined by market forces
without regulatory intervention, essentially
(b) Foreign Direct Investment
treating the Rupee like any international currency.
(c) Foreign Portfolio Investment
(d) External Commercial Borrowings
64. Consider the following statements: [2002]
[email protected]
Solution 62. Correct Option: (b) (1) The World Intellectual Property Organisation
Option (b) is correct (WIPO) is a specialised agency of United Nations
System of Organisations
• Foreign Direct Investment (FDI) is defined as
direct management ownership in one country's (2) WIPO has its headquarters at Rome
business by an entity based in another country.
(3) The Trade Related Aspects of Intellectual
• FDI distinguishes itself from foreign portfolio Property Rights (TRIPS) Agreement is binding on all
investment by9654469135
offering direct control, often WTO members
involving management participation, joint ventures,
technology transfer, and expertise. (4) Least developed country members of WTO are
not required to apply for the provisions of TRIPS
Agreements for a period of 20 years from the
63. Consider the following statements: [2002] general date of application of the Agreement

Full convertibility of the rupee may mean Which of these statements are correct?

(1) its free float with other international currencies. (a) 1, 2, 3 and 4
(b) 2, 3 and 4
(2) its direct exchange with any other international (c) 1, 2 and 4
currency at any prescribed place inside and outside
(d) 1 and 3
the country.

Solution 64. Correct Option: (d)

https://www.meritnest.com Page 31
Economy PYQs 3 of 5 External Sector, International Institutions

Option (d) is correct. 1. Brussels


9654469135
2. Geneva
• The World Intellectual Property Organization
(WIPO) is one of the 15 specialized agencies of the 3. Paris
United Nations, operational since 1967.
4. Vienna
• Its mandate is to promote and protect intellectual
Codes:
property (IP) worldwide through cooperation with
[email protected]
countries and international organizations. ABCD
(a) 1 2 4 3
• WIPO is headquartered in Geneva, Switzerland.
(b) 4 2 1 3
• The agency's activities include hosting forums for (c) 4 2 1 3
IP rules and policy discussion, providing services for (d) 4 2 1 3
IP registration and protection in various countries,
resolving transboundary IP disputes, and
Solution 65. Correct Option: (d)
connecting IP systems with uniform standards.
Option (d) is correct.
• WIPO serves as a reference database on IP
matters, offering reports and statistics on global • The International Atomic Energy Agency (IAEA)
and country-specific IP protection and innovation. promotes the peaceful use of nuclear energy and
aims to prevent its use for military purposes,
• In 2021, WIPO launched the Young Experts including nuclear weapons, established on 29 July
Program (YEP) to develop the next generation of IP 1957.
leaders.
• The IAEA is headquartered in Vienna, Austria.
• WIPO has 193 member states, including 190 UN
member states, the Cook Islands, Holy See, and • The International Telecommunication Union
Niue, with Palestine holding permanent observer (ITU), the oldest UN agency, focuses on information
status. and communication technologies, established on 17
May 1865 as the International Telegraph Union.
• The only non-members recognized by the UN are
the Federated States of Micronesia, Palau, and • The ITU is headquartered in Geneva, Switzerland.
South Sudan.
• The European Council (informally EUCO) outlines
• The World Trade Organization's Agreement on the political direction and priorities of the European
Trade-Related Aspects of Intellectual Property Union, composed
oslo of EU member states' heads of
Rights (TRIPS) integrates intellectual property rules state or government, the President of the European
into the multilateral trading system, establishing Council, and the President of the European
minimum standards for IP regulation applicable to Commission.
nationals of other WTO member nations.
• The European Council was formalized as an
institution in 2009 with the Treaty of Lisbon.
65. Match List-I (Organisation) with List-II
(Headquarters) and select the correct answer
• The Organisation for Economic Co-operation and
using the codes given below the Lists: [2002]
Development (OECD) is an intergovernmental
economic organization with 38 member countries,
List-I (Organisation)
founded in 1961 to stimulate economic progress
(A) International Atomic Energy Agency and world trade.
(B) International Telecommunication Union
• The OECD is headquartered in Paris, France,
(C) Council of the European Union serving as a forum for countries committed to
democracy and the market economy.
(D) Organisation for Economic Cooperation and
Development
66. The Asia Pacific Economic Cooperation Forum
List-II (Headquarters) was held in the year 2001 in: [2002]

https://www.meritnest.com Page 32
Economy PYQs 3 of 5 External Sector, International Institutions

offering banking and financial services. It was


(a) Bangkok
established in 1964 as the Industrial Development
(b) Hongkong Bank of India, a development finance institution
(c) Jakarta aimed at the industrial sector.
oslo
(d) Shanghai9654469135
• The National Bank for Agriculture and Rural
Development (NABARD) serves as a regulatory
Solution 66. Correct Option: (d)
body for regional rural banks and apex cooperative
Option (d) is correct. banks in India, under the Ministry of Finance,
Government of India. NABARD focuses on policy,
• APEC 2001 was held in Shanghai, China. planning, and operations in credit for agriculture
and other economic activities in rural areas. It was
established based on the recommendations of the
[email protected]
67. Consider the following financial institution of
B. Sivaramman Committee (by Act 61, 1981 of
India: [2002]
Parliament) on 12 July 1982.
(1) Industrial Finance Corporation of India (IFCI)
68. Assertion (A): Ceiling on foreign exchange for a
(2) Industrial Credit and Investment Corporation of host of current account transaction heads was
India (ICICI) lowered in the year 2000.

(3) Industrial Development Bank of India (IDBI) Reason (R): There was a fall in foreign currency
assets also. [2001]
(4) National Bank for Agriculture and Rural
(a) Both A and R are individually true, and R is the
Development (NABARD)
correct explanation of A
The correct chronological sequence of the (b) Both A and R are individually true, but R is NOT a
establishment of these institutions is: correct explanation of A
(c) A is true, but R is false
(a) 1, 2, 3, 4
(d) A is false, but R is true
(b) 2, 3, 4, 1
(c) 3, 4, 1, 2
Solution 68. Correct Option: (a)
(d) 4, 1, 2, 3
Option (a) is correct
Solution 67. Correct Option: (a)
• The Foreign Exchange Management Act (FEMA)
Option (a) is correct. defines 'Capital Account Transaction' as any
transaction that alters the assets or liabilities
• IFCI, previously known as the Industrial Finance outside India of persons residing in India or inside
Corporation of India, operates under the Ministry India of persons residing outside India.
of Finance, Government of India. It was established
in 1948 as a statutory corporation and is now a • In 2000, the ceiling on foreign exchange for
company listed on the BSE and NSE. various current account transactions was lowered.

• IFCI has seven subsidiaries and one associate. • During the first seven months of 2000-01, India's
foreign currency assets declined by about US $2.96
• ICICI Bank was founded by the Industrial Credit billion, reflecting the Reserve Bank of India's
and Investment Corporation of India (ICICI) in 1994 intervention in the foreign exchange market due to
in Vadodara. The parent company was formed in a surge in oil import bills.
1955 as a joint venture of the World Bank, India’s
public-sector banks, and public-sector insurance
69. Consider the following organisations: [2001]
companies to provide project financing to the
Indian industry.
I. International Bank for Reconstruction and
Development
• The IDBI Bank Limited, also known as IDBI Bank
or IDBI, is an Indian private sector bank and a
II. International Finance Corporation
subsidiary of the Life Insurance Corporation (LIC),

https://www.meritnest.com Page 33
Economy PYQs 3 of 5 External Sector, International Institutions

III. International Fund for Agricultural


Option (b) is correct.
Development
• The World Trade Organization (WTO) is a global
IV. International Monetary Fund
international organization dealing with trade rules
between nations, aiming to ensure trade flows
Which of these are the agencies of the United
smoothly, predictably, and freely.
Nations?
(a) I and II • The 1999 WTO Ministerial Conference in Seattle
(b) II and III was prematurely ended due to protests by
anti-globalization activists, known as the Battle of
(c) III and IV
Seattle.
(d) I, II, III and IV
• The 1999 ministerial meeting failed in part
Solution 69. Correct Option: (d) because it attempted to link trade with labor and
environmental issues, not due to terrorism-related
Option (d) is correct.
matters.
• All four mentioned entities are specialized
agencies of the United Nations, operating
[email protected] 71. Match List I with List II and select the correct
autonomously in collaboration with the UN. answer using the codes given below the Lists:
[1999]
• The International Fund for Agricultural
Development (IFAD) is a specialized agency of the List-I
UN focusing on poverty and hunger in rural areas of
I. WTO
developing countries, unique in its dedication to
rural economies and food security. II. IDA
III. IMF
• The International Monetary Fund (IMF), with 190
member countries, aims to promote global IV. IBRD
monetary cooperation, financial stability,
international trade, sustainable economic growth, List-II
and poverty reduction. It was established in 1944
A) Provides loans to address short-term balance of
and commenced operations on 27 December 1945.
payment problems
B) Multi-lateral trade negotiation body
70. Consider the following statement: [2000]
C) Sanction of soft loans
The Ministerial Meeting of the WTO held in
D) Facilitating lending and borrowing for
December 1999 was unsuccessful because it
reconstruction and development
attempted to link trade with:
Codes:
I. labour-related issues.
(a) I-B, II-C, III-D, IV-A 9654469135
II. environment related issues. (b) I-B, II-C, III-A, IV-D
(c) I-C, II-B, III-D, IV-A
III. terrorism related issues.
(d) I-C, II-B, III-A, IV-D
IV. debt-related issues.
Solution 71. Correct Option: (b)
Which of these statements are correct?
Option (b) is correct.
(a) I, III and IV
(b) I and II • The World Trade Organization (WTO) provides a
framework for negotiating trade agreements and
(c) II and III
facilitating trade in goods, services, and intellectual
(d) II and IV property among participating countries.
oslo
Solution 70. Correct Option: (b)

https://www.meritnest.com Page 34
Economy PYQs 3 of 5 External Sector, International Institutions

• The International Development Association (IDA) • Attracting Foreign Direct Investment (FDI) is an
is an international financial institution that offers example of how economic policies can impact
concessional loans and grants to the world's currency valuation.
poorest developing countries.
73. Capital Account Convertibility of the Indian
• The International Monetary Fund (IMF) works to
Rupee implies [1998]
strengthen global monetary cooperation, secure
financial stability, facilitate international trade, (a) that the Indian Rupee can be exchanged by the
promote high employment and sustainable authorised dealers for travel
economic growth, and reduce poverty worldwide. (b) that the Indian Rupee can be exchanged for any
major currency for the purpose of trade in goods
• The International Bank for Reconstruction and and services oslo
Development (IBRD) provides loans to
(c) that the Indian Rupee can be exchanged for any
middle-income developing countries for
major currency for the purpose of trading financial
reconstruction and development purposes.
assets
(d) None of the above
72. Consider the following statements: [1998]
Solution 73. Correct Option: (b)
The price of any currency in international market is
decided by the Option (b) is correct.

(I) World Bank. • Capital account convertibility signifies the


freedom to conduct investment transactions
(II) demand for goods/services provided by the without restrictions.
country concerned.
• It implies no restrictions on converting rupees
(III) stability of the government of the concerned into foreign currency for acquiring foreign assets by
country. an Indian resident.

(IV) economic potential of the country. • Full convertibility of the rupee would mean the
exchange rate would be determined by market
Of these statements: factors without regulatory intervention.
[email protected]
(a) I, II, III and IV are correct
• Full convertibility implies no limits on capital
(b) II and III are correct inflows or outflows for various purposes, including
(c) III and IV are correct investments and remittances.
(d) I and IV are correct
74. Assertion (A): The United States of America
Solution 72. Correct Option: (b) has threatened to ask the World Trade
Organisation (WTO) to apply sanctions against the
Option (b) is correct.
developing countries for the non-observance of
9654469135
ILO conventions.
• The World Bank does not influence the price of
any currency.
Reason (R): The United States of America itself has
adopted and implemented those ILO conventions.
• Economic potential is a futuristic aspect and does
[1998]
not directly link to the current price of a currency.
(a) Both A and R are true, and R is the correct
• The demand for goods and services reflects over explanation of A
foreign exchange reserves and offers a better way (b) Both A and R are true, but R is NOT a correct
to determine a currency's value. explanation of A
(c) A is true, but R is false
• The stability of a country is a major factor for its
currency's price as it reflects the vision and (d) A is false, but R is true
objectives of its economic policy.
Solution 74. Correct Option: (b)

https://www.meritnest.com Page 35
Economy PYQs 3 of 5 External Sector, International Institutions

Option (b) is correct. • Hawala is an informal value transfer system not


based on the movement of cash or wire transfers
• Labour standards are currently not governed by between banks.
WTO rules and disciplines. However, some member
governments, particularly in Europe and North • It operates outside traditional banking and
America, believe the WTO should address labour remittance systems, offering anonymity in
standards to enhance public confidence in the WTO transactions.
and the global trading system. [email protected]
• Hawala's network of money brokers honors and
• These governments advocate for the performs the transfer of funds without keeping
consideration of rights such as freedom to bargain official records.
collectively, freedom of association, elimination of
discrimination in the workplace, and the elimination • Hawala is gaining traction in financial technology,
of workplace abuse, including forced and certain providing access to money transfers for the
types of child labour, within the WTO framework. unbanked and underbanked populations.

• There is a consensus among WTO members on a


76. Consider the following items imported by
set of "core" internationally recognized standards:
India: [1996]
freedom of association, no forced labour, no child
labour, and no discrimination at work, including
(I) Capital goods
gender discrimination.
(II) Petroleum
• The United States and the International Labour
Organization (ILO) share common interests in
(III) Pearls and precious stones
promoting universal human rights through better
global living and working conditions, pledging to
(IV) Chemicals
instill respect for democratic principles.
(V) Iron and Steel
• The USA has passed several laws reflecting these
commitments, including the Tripartite Consultation
The correct sequence of the decreasing order of
(International Labour Standards) Act in 1988, the
these items, in terms of value, is
Abolition of Forced Labour Convention in 1991,
and the Worst Forms of Child Labour Convention in (a) I, II, III, IV, V
1999, which align with, but are specific to, the USA's (b) I, II, IV, III, V
context rather than directly mirroring ILO
(c) II, I, III, IV, V
conventions.
(d) II, I, IV, V, III

75. Hawala transactions relate to payments [1996]


oslo Solution 76. Correct Option: (a)
(a) received in rupees against overseas currencies
Option (a) is correct.
and vice-versa without going through the official
channels
• The provided diagram illustrates the order of
(b) received for sale/transfer of shares without commodities exported during the year 1994-95.
going through the established stock exchanges
(c) received as commission for services rendered to
77. Which of the following were the aims behind
overseas investors/buyers/sellers in assisting them
the setting up of the World Trade Organization?
to get over the red tape and/or in getting
[1996]
preferential treatment
(d) made to political parties or to individuals for I. Promotion of free trade and resource flows across
meeting election expenses countries.
9654469135

Solution 75. Correct Option: (a) II. Protection of intellectual property rights.
Option (a) is correct.
III. Managing balanced trade between different
countries.

https://www.meritnest.com Page 36
Economy PYQs 3 of 5 External Sector, International Institutions

IV. Promotion of trade between the former East


Codes:
block countries and the western world.
(a) II and IV are correct
Select the correct answer by using the codes given (b) I, II and III are correct
below:
(c) I, II and IV are correct
Code: (d) III and IV are correct
[email protected]
(a) I, II, III and IV
Solution 78. Correct Option: (b)
(b) I and II
Option (b) is correct.
(c) II and III
(d) I and IV • International agencies like the World Trade
Organization, International Monetary Fund, World
Solution 77. Correct Option: (b) Bank, and other United Nations agencies offer
technical assistance, grants, and loans to countries.
Option (b) is correct.
• The International Development Agency provides
• The World Trade Organization (WTO) is the
soft loans to third-world countries to address
world's largest international economic
poverty and related social issues.
organization, with 164 member states representing
over 98% of global trade and GDP.
• The Food and Agriculture Organization offers
food assistance to countries in need.
• It provides a framework for negotiating trade
agreements aimed at reducing or eliminating tariffs,
quotas, and other trade restrictions. 79. The new Exim Policy announced in 1992, is for
a period of [1995]
• The WTO administers independent dispute
(a) 3 years
resolution to enforce trade agreements and resolve
disputes among countries. (b) 4 years
(c) 7 years
• It prohibits discrimination between trading (d) 5 years
partners but allows exceptions for environmental
protection, national security, and other important
Solution 79. Correct Option: (d)
objectives.
Option (d) is correct.
• The primary goal of the WTO is to facilitate open
trade for the benefit of all. • The Export-Import policy (EXIM Policy) is
announced under the Foreign Trade (Development
oslo and Regulation) Act 1992.
78. Consider the following statements: [1996]
• It aims to reflect the extent of regulations or
Most international agencies which fund
liberalization of foreign trade and measures for
development programmes in India on
export promotion.
intergovernmental bilateral agreements, mainly
provide:
• The EXIM Policy is designed for a period of 5
years, indicating the government's approach to
I. Technical assistance.
foreign trade and investment.
II. Soft loans which are required to be paid back
• It is a significant part of India's trade and
with interest.
economic policy framework, focusing on enhancing
the country's export capabilities and liberalizing
III. Grants, not required to be paid back.
9654469135
imports to support economic growth.
IV. Food assistance to alleviate poverty.
80. Which of the following constitute the World
Choose the correct answer from the codes given Bank? [1995]
below:

https://www.meritnest.com Page 37
Economy PYQs 3 of 5 External Sector, International Institutions

I. International Bank for Reconstruction and III. India’s share in the world trade is likely to triple
Development by the year 2000 A.D.

II. International Finance Corporation Select the correct answer using the codes given
oslo
9654469135 below:
III. International Development Association
Codes:
IV. International Monetary Fund
(a) I, II and III
Choose the correct answer from the codes given (b) I and II
below: (c) I and III
(d) II and III
Codes: [email protected]

(a) I, II and III Solution 81. Correct Option: (d)


(b) I and II Option (d) is correct.
(c) III and IV
(d) I, II, III and IV • India's export share in world trade has seen a
significant increase recently.
Solution 80. Correct Option: (a)
• The country's exports as a percentage of world
Option (a) is correct. exports rose from 0.48% in the 1980s to 0.56%
during 1991-96, and further to 0.65% during
• The International Bank for Reconstruction and 1996-2002.
Development (IBRD), known as the World Bank,
was founded in 1944 and has grown into a group of • The ratio reached 0.71% in 2000-01, marking the
five development institutions. highest level since the 1970s.

• The World Bank Group includes the IBRD, • Despite these improvements, India's share in
International Development Association (IDA), world exports remains relatively low compared to
International Finance Corporation (IFC), other major trading Asian countries such as China,
Multilateral Guarantee Agency (MIGA), and Malaysia, Thailand, Singapore, Korea, and
International Centre for the Settlement of Indonesia.
Investment Disputes (ICSID).

• The IBRD offers loans, credits, and grants; the IDA


provides low or no-interest loans to low-income
countries; the IFC supports private sector
development; MIGA insures against political risks;
and ICSID settles investment disputes.

• The World Bank periodically releases the Global


Economic Prospects report.

81. Which of the following benefits are likely to


accrue to India from the World Trade
Organisation? [1995]

I. India’s share in the world trade is to go up from


the present 600 million US dollars to 5 billion US
dollars by 2000 A.D.

II. It will help boost exports of agricultural


commodities from India.

https://www.meritnest.com Page 38

You might also like